Expressing geometrically the nth roots of a complex number on a circle

Click For Summary
SUMMARY

The discussion centers on the geometric representation of the nth roots of a complex number, specifically proving that z^{1/n} corresponds to n equally spaced points on the circle defined by x^2 + y^2 = |z|^2. Participants express confusion regarding the modulus of the roots, noting that the roots, which have a modulus of |z|^{1/n}, do not lie on the circle of radius |z|. The consensus is that the original question is flawed, as the roots should lie on a circle of radius |z|^{1/n} instead.

PREREQUISITES
  • Understanding of complex numbers and their polar representation
  • Familiarity with the concept of modulus and argument of a complex number
  • Knowledge of De Moivre's Theorem
  • Basic trigonometry for interpreting angles in the complex plane
NEXT STEPS
  • Study the geometric interpretation of complex numbers in polar coordinates
  • Learn about De Moivre's Theorem and its applications in finding roots of complex numbers
  • Explore the properties of complex roots and their distribution on the complex plane
  • Investigate the implications of modulus in complex number transformations
USEFUL FOR

Mathematicians, students studying complex analysis, and anyone interested in the geometric properties of complex numbers and their roots.

5hassay
Messages
81
Reaction score
0

Homework Statement



Let z \in \mathbb{C}. Prove that z^{1/n} can be expressed geometrically as n equally spaced points on the circle x^2 + y^2 = |z|^2, where |z|=|a+bi|=\sqrt{a^2 + b^2}, the modulus of z.

Homework Equations



//

The Attempt at a Solution



My problem is that I am confused with the question.

I understand that

z^{1/n} = |z|^{1/n} (\cos(\frac{\theta}{n} + \frac{2k\pi}{n}) + \sin(\frac{\theta}{n} + \frac{2k\pi}{n}) i)

for k=0,1,\ldots,n-1, where \theta=\mathrm{Arg}(z).

Yet, my confusion is this: Each above complex number we get for each k will produce a complex number with modulus |z|^{1/n}. Because the given circle is of radius |z|, I cannot see how any of these complex numbers can lie on it for a arbitrary complex number z.

Where is my confusion lying, or is there a mistake in this question? I had asked the question giver, but they explained that the modulus didn't matter for plotting on such circle, which didn't make sense...

Thanks you.
 
Physics news on Phys.org
5hassay said:

Homework Statement



Let z \in \mathbb{C}. Prove that z^{1/n} can be expressed geometrically as n equally spaced points on the circle x^2 + y^2 = |z|^2, where |z|=|a+bi|=\sqrt{a^2 + b^2}, the modulus of z.

Homework Equations



//

The Attempt at a Solution



My problem is that I am confused with the question.

I understand that

z^{1/n} = |z|^{1/n} (\cos(\frac{\theta}{n} + \frac{2k\pi}{n}) + \sin(\frac{\theta}{n} + \frac{2k\pi}{n}) i)

for k=0,1,\ldots,n-1, where \theta=\mathrm{Arg}(z).

Yet, my confusion is this: Each above complex number we get for each k will produce a complex number with modulus |z|^{1/n}. Because the given circle is of radius |z|, I cannot see how any of these complex numbers can lie on it for a arbitrary complex number z.

Where is my confusion lying, or is there a mistake in this question? I had asked the question giver, but they explained that the modulus didn't matter for plotting on such circle, which didn't make sense...

Thanks you.

I agree with you. E.g. If you put z=2 and n=2 then the values of 2^(1/2) are sqrt(2) and -sqrt(2). They lie on the circle of radius sqrt(2) not the circle of radius 2. The question is bad.
 
Dick said:
I agree with you. E.g. If you put z=2 and n=2 then the values of 2^(1/2) are sqrt(2) and -sqrt(2). They lie on the circle of radius sqrt(2) not the circle of radius 2. The question is bad.

Yeah, that's what I am thinking. Thanks for the reply
 
Question: A clock's minute hand has length 4 and its hour hand has length 3. What is the distance between the tips at the moment when it is increasing most rapidly?(Putnam Exam Question) Answer: Making assumption that both the hands moves at constant angular velocities, the answer is ## \sqrt{7} .## But don't you think this assumption is somewhat doubtful and wrong?

Similar threads

  • · Replies 22 ·
Replies
22
Views
973
  • · Replies 17 ·
Replies
17
Views
2K
  • · Replies 6 ·
Replies
6
Views
2K
  • · Replies 6 ·
Replies
6
Views
2K
Replies
8
Views
3K
Replies
7
Views
4K
  • · Replies 14 ·
Replies
14
Views
2K
  • · Replies 7 ·
Replies
7
Views
2K
  • · Replies 4 ·
Replies
4
Views
3K